2
$\begingroup$

Suppose a bounded sequence $(x_n)$ converges to $x$ in the Cesaro sense (i.e., $\frac{1}{n}(x_1 + x_2 + \dots + x_n)\rightarrow x$) in a separable Hilbert space $H$. How to prove that some subsequence $(x_{n_k})$ converges weakly to $x$?

$\endgroup$
3
  • 1
    $\begingroup$ Reminds me of Banach-Saks theorem, which goes the other way. $\endgroup$ Jul 19, 2010 at 15:07
  • $\begingroup$ Indeed, looks like the exercise might've been intended the other way. For the record: the proof of Banach-Saks can be found in books.google.com/… $\endgroup$ Jul 19, 2010 at 16:53
  • 1
    $\begingroup$ Whatever the corrected question is supposed to be, it almost certainly is not research level. $\endgroup$
    – Zen Harper
    Jul 19, 2010 at 18:13

1 Answer 1

8
$\begingroup$

If we take $x_n = (-1)^n x$ then $x_n$ converges to $0$ in Cesaro sence. But no subsequence of $x_n$ converges weakly to $0$. $x_n$ is also a bounded sequence. Hence your statements seems wrong.

$\endgroup$
3
  • $\begingroup$ Thanks. It was an exercise that I couldn't solve, so I assumed it should hold. $\endgroup$ Jul 19, 2010 at 15:13
  • 5
    $\begingroup$ @Kestutis Cesnavicius: A good habit to be in is that if you cannot prove a statement, try to break it. Either you succeed in falsifying the statement, or you gain more understanding of why the statement should be true. $\endgroup$ Jul 19, 2010 at 17:57
  • 2
    $\begingroup$ Kestutis, may be it was asked (or they wanted to ask) to prove that some subsequence weakly converges, but not necessary to $x$? Which is not very deep, too, but it is true at least. $\endgroup$ Jul 19, 2010 at 18:06

Your Answer

By clicking “Post Your Answer”, you agree to our terms of service and acknowledge you have read our privacy policy.

Not the answer you're looking for? Browse other questions tagged or ask your own question.